Intégrale

Bonjour
Calculer $$\int_{0}^{\pi/2}\frac{x\ln(1-\sin(x))}{\sin(x)}dx$$ Un éclairage je vous remercie.

Réponses

  • Bonjour,

    Très difficile : d’abord une IPP pour se débarrasser du $x$, puis CDV avec $y=\tan(x/2)$ et quelques manipulations idoines pour utiliser des symétries (sur les bornes...).
    C’est une idée plutôt qu’une certitude, mais j’ai confiance.
    On peut aussi développer la dernière intégrale en puissances de sinus...
  • Il me semble que l'intégrale qu'il ait est demandé de calculer ici vaut 2 fois celle-ci (au signe près):
    http://www.les-mathematiques.net/phorum/read.php?4,1572370,1663706#msg-1663706

    PS:
    J'ai un doute tout de même.

    PS2:
    Sacrée coïncidence.
  • Une idée On pose
    $\displaystyle I(a)=\int_{0}^{\pi/2}\frac{x\ln(1+a\sin(x))}{\sin(x)}dx~$ alors $~\displaystyle I'(a)=\int_{0}^{\pi/2}\frac{x}{1+a\sin(x)}$
    Ce logiciel https://www.integral-calculator.com/ permet de donner une primitive de $\displaystyle \frac{x}{1+a\sin(x)}$ en "step by step" mais j'ai la flemme de continuer
    Le 😄 Farceur


  • Sans le x c'est faisable niveau prépas on développe
    ln(1-t) puis on permute somme et intégrale et on utilise
    intégrale de Wallis .

    Mais avec le x infaisable en prépas.
  • @etanche
    Sans le x la méthode que j'ai donné marche très bien. Confirmes-tu?
    Le 😄 Farceur


  • Il y a un problème en zéro et ${\pi/2}$ déja...Sinon, la méthode de Gébrane marche
  • Gebrane:

    Dit autrement:

    On sait que pour $-1<u<1$:

    $\displaystyle \ln(1+u)=\int_0^1 \frac{u}{tu-1}\,dt$

    donc:

    $\begin{align}I&=\int_{0}^{\pi/2}\frac{x\ln(1-\sin(x))}{\sin(x)}\,dx\\
    &=\int_{0}^{\pi/2}\left(\int_0^1 \frac{x}{t\sin(x)-1}\,dt\right)\,dx\\
    &=\int_0^1 \left(\left[\frac{2x}{\sqrt{1-t^2}}\arctan\left(\frac{t-\tan\left(\frac{x}{2}\right)}{\sqrt{1-t^2}}\right)\right]_{0}^{\pi/2}-\int_0^1 \frac{2}{\sqrt{1-t^2}}\arctan\left(\frac{t-\tan\left(\frac{x}{2}\right)}{\sqrt{1-t^2}}\right)\,dx\right)\,dt\\
    &=-\frac{\pi}{2}\int_0^1 \frac{2}{\sqrt{1-t^2}}\arctan\left(\sqrt{\frac{1-t}{1+t}}\right)\,dt-\int_0^1 \left(\int_{0}^{\pi/2} \frac{2}{\sqrt{1-t^2}}\arctan\left(\frac{t-\tan\left(\frac{x}{2}\right)}{\sqrt{1-t^2}}\right)\,dx\right)\,dt\\
    &=-\pi\left[\arctan^2\left(\sqrt{\frac{1-t}{1+t}}\right)\right]_0^1-\int_0^1 \left(\int_{0}^{\pi/2} \frac{2}{\sqrt{1-t^2}}\arctan\left(\frac{t-\tan\left(\frac{x}{2}\right)}{\sqrt{1-t^2}}\right)\,dx\right)\,dt\\
    &=-\frac{\pi^3}{16}-\int_0^1 \left(\int_{0}^{\pi/2} \frac{2}{\sqrt{1-t^2}}\arctan\left(\frac{t-\tan\left(\frac{x}{2}\right)}{\sqrt{1-t^2}}\right)\,dx\right)\,dt\\
    \end{align}$

    Il reste à calculer l'intégrale double. Je connais le résultat mais je ne sais pas le démontrer pour le moment.


    PS:
    Si cela peut aider au calcul de l'intégrale initiale à calculer. Elle vaut $-\dfrac{\pi^3}{8}$.

    PS2:
    J'aime beaucoup quand Wolfy me gratifie d'un tel message:

    https://www.wolframalpha.com/input/?i=integrate+atan((x-tan(t/2))/sqrt(1-x^2))/sqrt(1-x^2),x
  • Zorg69:

    Je suis curieux de voir le détail des calculs.
  • Je sais comment finir (sauf erreur). Stay tuned folks !
  • Une autre intégrale qui donne $\pi^3/8$

    $\int_{0}^{+\infty}\frac{x^2}{\cosh(x)}dx=\pi^3/8$
  • J'espère qu'il n'y a pas (trop) d'erreurs.

    Si on fait le changement de variable $y=\sin x$,

    $\begin{align}I&=\int_{0}^{\pi/2}\frac{x\ln(1-\sin(x))}{\sin(x)}dx\\
    &=\int_0^1 \frac{\ln(1-x)\arcsin x}{x\sqrt{1-x^2}}\end{align}$

    Je me propose de calculer l'intégrale double d'un message précédent.

    $\begin{align}J&=\int_0^1 \left(\int_{0}^{\pi/2} \frac{2}{\sqrt{1-t^2}}\arctan\left(\frac{t-\tan\left(\frac{x}{2}\right)}{\sqrt{1-t^2}}\right)\,dx\right)\,dt\\
    &=2\int_{0}^{\pi/2}\left(\left[\arctan\left(\frac{t-\tan\left(\frac{x}{2}\right)}{\sqrt{1-t^2}}\right)\arcsin t\right]_0^1-\int_0^1 \frac{\left(1-t\tan\left(\frac{x}{2}\right)\right)\arcsin t}{\left(\tan^2\left(\frac{x}{2}\right)-2t\tan\left(\frac{x}{2}\right)+1\right)\sqrt{1-t^2}}\,dt \right)\,dx\\
    &=2\int_{0}^{\pi/2}\left(\frac{\pi^2}{4}-\int_0^1 \frac{\left(1-t\tan\left(\frac{x}{2}\right)\right)\arcsin t}{\left(\tan^2\left(\frac{x}{2}\right)-2t\tan\left(\frac{x}{2}\right)+1\right)\sqrt{1-t^2}}\,dt \right)\,dx\\
    &=\frac{\pi^3}{8}-\int_0^1 \left[\frac{\left(tx-\ln\left(\tan^2\left(\frac{x}{2}\right)-2t\tan\left(\frac{x}{2}\right)+1\right)+\ln\left(1+\tan^2\left(\frac{x}{2}\right)\right)\right)\arcsin t}{t\sqrt{1-t^2}}\right]_{x=0}^{x=\pi/2}\,dt\\
    &=\frac{\pi^3}{8}-\int_0^1 \frac{\arcsin t\left(\frac{t\pi}{2}-\ln(1-t)\right)}{t\sqrt{1-t^2}}\,dt\\
    &=\frac{\pi^3}{8}+I+\frac{1}{2}\pi\int_0^1 \frac{\arcsin t}{\sqrt{1-t^2}}\,dt\\
    &=\frac{\pi^3}{8}+I+\frac{1}{4}\pi\Big[\arcsin^2 x\Big]_0^1\\
    &=\frac{3\pi^3}{16}+I\\
    \end{align}$

    D'où:

    $\begin{align}I&=-\frac{\pi^3}{16}-\left(\frac{3\pi^3}{16}+I\right)\\
    &=-\frac{\pi^3}{4}-I
    \end{align}$

    Ainsi,

    $\displaystyle \boxed{I=-\dfrac{\pi^3}{8}}$

    PS:
    Comme déjà indiqué j'ai une deuxième méthode qui fonctionne mais elle a l'inconvénient d'utiliser le fait que $\zeta(2)=\dfrac{\pi^2}{6}$.
  • Etanche:

    On intègre par parties,

    $\begin{align}K&=\int_0^{\infty}\frac{x^2}{\cosh x}\,dx\\
    &=\left[-2x^2\arctan\left(\text{e}^{-x}\right)\right]_0^{\infty}+4\int_0^{\infty}x\arctan\left(\text{e}^{-x}\right)\,dx\\
    &=4\int_0^{\infty}x\arctan\left(\text{e}^{-x}\right)\,dx
    \end{align}$

    On fait le changement de variable $y=\text{e}^{-x}$,

    $\begin{align}K&=-4\int_0^1 \frac{\ln x\arctan x}{x}\,dx\end{align}$

    Si on développe en série l'intégrande et qu'on intègre terme à terme la série obtenue on obtient une série d'Euler:

    $\displaystyle -\int_0^1 \frac{\ln x\arctan x}{x}\,dx=\sum_{n=0}^{\infty} \frac{(-1)^n}{(2n+1)^3}$

    et on sait que cette somme vaut $\dfrac{\pi^3}{32}$.

    PS:
    La série est aussi égale à $\beta(3)$ , ici $\beta$ est la fonction bêta de Dirichlet (cf. https://fr.wikipedia.org/wiki/Fonction_bêta_de_Dirichlet )

    PS2:
    Ce serait plus sympathique de pouvoir transformer (intégrations par parties, changements de variable) l'intégrale K en l'intégrale I.

    PS3:

    Une méthode possible du calcul de $\displaystyle \int_0^1 \frac{\ln x\arctan x}{x}\,dx$.

    On intègre par parties,

    $\begin{align}\int_0^1 \frac{\ln x\arctan x}{x}\,dx=\left[\ln^2 x\arctan x\right]_0^1-\int_0^1 \ln x\left(\frac{\arctan x}{x}+\frac{\ln^2 x}{1+x^2}\right)\,dx\end{align}$

    donc,

    $\begin{align}\int_0^1 \frac{\ln x\arctan x}{x}\,dx=-\frac{1}{2}\int_0^1 \frac{\ln^2 x}{1+x^2}\,dx\end{align}$

    On considère la fonction $\displaystyle F(s)=\int_0^1 \frac{x^s}{1+x^2}\,dx$ avec $s\geq 0$

    On a, sauf erreur, $\displaystyle F^{\prime\prime}(0)=\int_0^1 \frac{\ln^2 x}{1+x^2}\,dx$

    $F(s)$ peut s'exprimer comme fonction bêta d'Euler.
  • Et celle-ci $$\int_{0}^{\pi/2}\frac{x^2\ln(1-\sin(x))}{\sin(x)}dx$$
Connectez-vous ou Inscrivez-vous pour répondre.